Taylorreihe der Logarithmusfunktion

Satz 16K6

Gegeben sei die Logarithmusfunktion f(x)=ln(1+x)f(x)=\ln(1+x) mit dem Entwicklungspunkt x0=0x_0=0. Es gilt
ln(1+x)=k=1n(1)k1kxk+Rn(x)\ln(1+x)=\sum\limits_{k=1}^n \dfrac{(-1)^{k-1}}{k}x^k+R_n(x) (1)
wobei
Rn(x)=(1)nΘ1xn+1n+1R_n(x)=(-1)^n\Theta_1\dfrac{x^{n+1}}{n+1}, 0<Θ1<10<\Theta_1<1, 0x10\leq x\leq 1
Rn(x)=(1)nΘ2xn+11+xR_n(x)=(-1)^n\Theta_2\dfrac{x^{n+1}}{1+x} 0<Θ2<10<\Theta_2<1, 1<x<0-1<x<0
(Θ1,Θ2\Theta_1, \Theta_2 hängen von nn und xx ab.) Die Taylorreihe
ln(1+x)=k=1(1)k1kxk\ln(1+x)=\sum\limits_{k=1}^\infty \dfrac{(-1)^{k-1}}{k}x^k 1<x1 -1<x\leq 1
ist für 1<x<1-1<x<1 absolut konvergent und für x>1|x|>1 divergent.

Beweis

f(x)=ln(1+x)f(x)=\ln(1+x); f(x)=11+xf\, '(x)=\dfrac{1}{1+x}; f(x)=1(1+x)2 f\, ''(x)=-\dfrac{1}{(1+x)^2}; f(x)=2(1+x)3 f'''(x)=\dfrac{2}{(1+x)^3}; Allgemein: f(n)(x)=(n1)!(1)n11(1+x)n f^{(n)}(x)=(n-1)!(-1)^{n-1}\dfrac{1}{(1+x)^n} Einsetzen in Taylorformel: ln(1+x)=f(x) \ln(1+x)= f(x)=k=0nf(k)(0)k!+Rn =\sum\limits_{k=0}^n \dfrac{f^{(k)}(0)}{k!}+R_n=k=1n(k1)!(1)k11k!xk+Rn(x) = \sum\limits_{k=1}^n (k-1)!(-1)^{k-1}\dfrac{1}{k!}x^k+R_n(x)=k=1n(1)k1xkk+Rn(x) = \sum\limits_{k=1}^n (-1)^{k-1}\dfrac{x^k}{k}+R_n(x)
Untersuchung der Restglieder:

1. Fall

0x10\leq x\leq 1: Restglied von Lagrange: Rn(x)=f(n+1)(Θx)(n+1)!xn+1R_n(x)=\dfrac{f^{(n+1)}(\Theta x)}{(n+1)!}x^{n+1}=(1)n1(1+Θx)n+1xn+1n+1 =(-1)^n {\dfrac{1}{(1+\Theta x)^{n+1}}}\, \dfrac{x^{n+1}}{n+1} mit 0<Θ10<\Theta\leq 1. Wir setzen Θ1:=1(1+Θx)n+1\Theta_1:=\dfrac{1}{(1+\Theta x)^{n+1}} und es gilt 0<Θ1<10<\Theta_1<1 , denn 1+Θx11+\Theta x\geq 1. Daher folgt, 11+Θx1\dfrac{1}{1+\Theta x}\leq 1. Somit ist Rn(x)=(1)nΘ1xn+1n+1R_n(x)=(-1)^n\Theta_1\dfrac{x^{n+1}}{n+1} für alle 0x10\leq x\leq 1 und es folgt Rn(x)xn+1n+1n0|R_n(x)|\leq\dfrac{|x|^{n+1}}{n+1}\, \, \stackrel{n\rightarrow\infty}\longrightarrow 0limnRn(x)=0 \Rightarrow\lim_{n\rightarrow\infty} R_n(x)=0 für 0x10\leq x \leq 1.

2. Fall

1<x0-1<x\leq 0: Cauchysches Restglied: Rn(x)=f(n+1)(Θx)n!(1Θ)nxn+1R_n(x)= \dfrac{f^{(n+1)}(\Theta x)}{n!}(1-\Theta)^nx^n+1 für 0<Θ<1 0<\Theta <1 Rn(x)=(1)n1(1+Θx)n+1(1Θ)nxn+1R_n(x) = (-1)^n\dfrac{1}{(1+\Theta x)^{n+1}}(1-\Theta)^nx^{n+1}=(1)n(1Θ1+Θx)n1+x1+Θxxn+11+x = (-1)^n{\left( \dfrac{1-\Theta}{1+\Theta x} \right)^n \dfrac{1+x}{1+\Theta x}}\, \dfrac{x^{n+1}}{1+x} Wir setzen Θ2:=(1Θ1+Θx)n1+x1+Θx\Theta_2:= \left( \dfrac{1-\Theta}{1+\Theta x} \right)^n \dfrac{1+x}{1+\Theta x} Es gilt 0<Θ2<10<\Theta_2<1 , da 0<1Θ<1+Θx0<1-\Theta<1+\Theta x und 0<1+x<1+Θx0<1+x<1+\Theta x. Somit ist: Rn(x)=(1)nΘ2xn+11+x R_n(x)= (-1)^n\Theta_2\dfrac{x^{n+1}}{1+x} Rn(x)xn+11+xn0 \Rightarrow |R_n(x)|\leq\dfrac{|x|^{n+1}}{1+x}\, \stackrel{n\rightarrow\infty}\longrightarrow 0 limnRn(x)=0 \Rightarrow \lim_{n\rightarrow\infty} R_n(x)=0 für 1<x0 -1<x\leq 0 Beide Fälle liefern die Konvergenz der Taylorreihe. \qed

Folgerung 16K7

Für 1y<1\leq y<\infty gilt:
lny=2k=1n1(2k1)(y1y+1)2k1+Rn~(y) \ln y = 2\sum\limits_{k=1}^n \dfrac{1}{(2k-1)}\left( \dfrac{y-1}{y+1} \right)^{2k-1} +\widetilde{R_n}(y)
Rn~(y)=(Θ12n+1+Θ22(y+1))(y1y+1)2n+1 \widetilde{R_n}(y) = \left( \dfrac{\Theta_1}{2n+1}+\dfrac{\Theta_2}{2}(y+1) \right) \left( \dfrac{y-1}{y+1} \right)^{2n+1} 0<Θ1,Θ2<1 0<\Theta_1,\Theta_2<1

Beweis

Sei 1y<1\leq y <\infty und x:=y1y+1x:=\dfrac{y-1}{y+1}. Es gilt 0x<10\leq x<1 und y=1+x1xy=\dfrac{1+x}{1-x}. ln(1+x)=k=12n(1)k1kxk+R2n(x) \ln(1+x) = \sum\limits_{k=1}^{2n}\dfrac{(-1)^{k-1}}{k}x^k+R_{2n}(x) ln(1x)=k=12n(1)k1k(x)k+R2n(x) \ln(1-x) = \sum\limits_{k=1}^{2n}\dfrac{(-1)^{k-1}}{k}(-x)^k+R_{2n}(-x) ln(1+x1x)=ln(1+x)ln(1x) \ln\left( \dfrac{1+x}{1-x} \right) = \ln(1+x)-\ln(1-x) =k=12n(1)k1kxk+R2n(x)k=12n(1)k1k(x)k+R2n(x) =\sum\limits_{k=1}^{2n}\dfrac{(-1)^{k-1}}{k}x^k+R_{2n}(x)- \sum\limits_{k=1}^{2n}\dfrac{(-1)^{k-1}}{k}(-x)^k+R_{2n}(-x) =k=12n(1)k1(1)kxk+(R2n(x)R2n(x)) = \sum\limits_{k=1}^{2n}\dfrac{(-1)^{k-1}-(-1)}{k}x^k+{(R_{2n}(x)-R_{2n}(-x))} =2k=1nx2k12k1+Rn~(x) = 2\sum\limits_{k=1}^n\dfrac{x^{2k-1}}{2k-1}+\widetilde{R_n}(x) mit Rn~:=(R2n(x)R2n(x))\widetilde{R_n}:=(R_{2n}(x)-R_{2n}(-x))
R2n(x)=(1)2nΘ1x2n+12n+1 R_{2n}(x)= (-1)^{2n}\Theta_1\dfrac{x^{2n+1}}{2n+1} R2n(x)=(1)2nΘ2(x)2n+11x R_{2n}(-x)= (-1)^{2n}\Theta_2\dfrac{(-x)^{2n+1}}{1-x} Rn~(x)=(Θ12n+1+Θ21x)x2n+1 \widetilde{R_n}(x)= \left( \dfrac{\Theta_1}{2n+1}+\dfrac{\Theta_2}{1-x} \right) x^{2n+1} Damit ist:
ln(y)=2k=1n12k1(y1y+1)2k1+Rn~(y){\ln(y)=2\sum\limits_{k=1}^n\dfrac{1}{2k-1}\left( \dfrac{y-1}{y+1} \right)^{2k-1} +\widetilde{R_n}(y)}
mit
Rn~=(Θ12n+1+Θ22(y+1))(y1y+1)2n+1{\widetilde{R_n}=\left( \dfrac{\Theta_1}{2n+1}+\dfrac{\Theta_2}{2} (y+1)\right)\left( \dfrac{y-1}{y+1} \right)^{2n+1}}
\qed

Spezialfälle

Für 1y<1\leq y<\infty gilt:
lny=2k=1n12k1(y1y+1)2k1+Rn~(y) \ln y=2\sum\limits_{k=1}^n \dfrac{1}{2k-1}\left(\dfrac{y-1}{y+1}\right)^{2k-1} +\widetilde{R_n}(y) (2)
Rn~(y)=(Θ12n+1+Θ22(y+1))(y1y+1)2n+1\widetilde{R_n}(y)=\left(\dfrac{\Theta_1}{2n+1}+\dfrac{\Theta_2}{2}(y+1)\right) \left(\dfrac{y-1}{y+1}\right)^{2n+1} 0<Θ1,Θ2<1 0<\Theta_1,\Theta_2<1
Für 0<y10<y\leq 1 nutzt man lny=ln(1y)\ln y=-\ln\left(\dfrac{1}{y}\right). Insbesondere
k=1(1)k1k=ln2=k=1n(1)k1k+Rn(1)\sum\limits_{k=1}^\infty \dfrac{(-1)^{k-1}}{k}=\ln 2=\sum\limits_{k=1}^n \dfrac{(-1)^{k-1}}{k}+R_n(1)
Rn(1)=(1)nΘ11n+1|R_n(1)|=|(-1)^n||\Theta_1|\dfrac{1}{n+1}1n+1n0 \leq\dfrac{1}{n+1} \stackrel{n\rightarrow\infty}\longrightarrow0
Genauigkeit auf sechs Stellen erhält man mit Sicherheit für n106n\geq 10^6 (schlechte Konvergenz), da Rn(1)1n+11n106n106|R_n(1)|\leq \dfrac{1}{n+1}\leq \dfrac{1}{n}\leq 10^{-6} \Rightarrow n\geq 10^6. Setzt man y=2y=2 in Gleichung (2) ein, erhält man ln2=2k=1n12k1(13)2k1+Rn~(2)\ln 2=2\sum\limits_{k=1}^n \dfrac{1}{2k-1}\left( \dfrac{1}{3} \right)^{2k-1}+\widetilde{R_n}(2) und
Rn~(2)=Θ12n+1+32Θ2(32)2n+1|\widetilde{R_n}(2)|=\left|\dfrac{\Theta_1}{2n+1}+\dfrac{3}{2}\Theta_2\right| \left( \dfrac{3}{2} \right)^{2n+1}(12n+1+32)(13)2n+1 \leq\left( \dfrac{1}{2n+1}+\dfrac{3}{2} \right) \left( \dfrac{1}{3} \right)^{2n+1}
n=6 n=6: R6~(2)(113+32)(13)13<106 |\widetilde{R_6}(2)|\leq \left( \dfrac{1}{13}+\dfrac{3}{2} \right) \left( \dfrac{1}{3} \right)^{13}<10^{-6}
n=10 n=10: R10~(2)(121+32)(13)21<1,51010|\widetilde{R_{10}}(2)|\leq \left( \dfrac{1}{21}+\dfrac{3}{2} \right) \left( \dfrac{1}{3} \right)^{21}<1,5\cdot 10^{-10}
 
 

Ein Mathematiker ist eine Maschine, die Kaffee in Theoreme verwandelt.

Paul Erdös

Copyright- und Lizenzinformationen: Diese Seite ist urheberrechtlich geschützt und darf ohne Genehmigung des Autors nicht weiterverwendet werden.
Anbieterkеnnzeichnung: Mathеpеdιa von Тhοmas Stеιnfеld  • Dοrfplatz 25  •  17237 Blankеnsее  • Tel.: 01734332309 (Vodafone/D2)  •  Email: cο@maτhepedιa.dе